Đến nội dung

Hình ảnh

[TOPIC] ÔN THI BẤT ĐẲNG THỨC $\boxed{\text{THPT CHUYÊN}}$ LỚP $10$ năm $2018-2019$

bất đẳng thức holder cosi bunhiacopxki

  • Chủ đề bị khóa Chủ đề bị khóa
Chủ đề này có 318 trả lời

#181
tr2512

tr2512

    Thượng sĩ

  • Thành viên
  • 272 Bài viết

Bài 81(VMO 2015): Cho a, b, c là các số thực dương. CMR:

$3(a^2+b^2+c^2)\geq (a+b+c)(\sqrt{ab}+\sqrt{bc}+\sqrt{ca})+(a-b)^2+(b-c)^2+(c-a)^2\geq (a+b+c)^2$

Ghê thật, VMO cơ à :D 

$ 3a^2+3b^2+3c^2 \ge (a+b+c)(\sqrt{ab}+\sqrt{bc}+\sqrt{ca})+(a-b)^2+(b-c)^2+(c-a)^2 $
$ \Leftrightarrow 3a^2+3b^2+3c^2 \ge (a+b+c)(\sqrt{ab}+s\sqrt{bc}+\sqrt{ca})+2a^2+2b^2+2c^2-2ab-2bc-2ca $
$ \Leftrightarrow (a+b+c)^2 \ge (a+b+c)(\sqrt{ab}+\sqrt{bc}+\sqrt{ca}) $ (Hiển nhiên đúng)
$ (a+b+c)(\sqrt{ab}+\sqrt{bc}+\sqrt{ca})+(a-b)^2+(b-c)^2+(c-a)^2 \ge (a+b+c)^2 $
Đặt $ x=\sqrt{a}; y=\sqrt{b}; z=\sqrt{c} $
$ \Rightarrow (x^2+y^2+z^2)(xy+yz+zx)+2x^4+2y^4+2z^4-2x^2y^2-2y^2z^2-2z^2x^2 \ge x^4+y^4+z^4+2x^2y^2+2y^2z^2+2x^2z^2 $
$ \Leftrightarrow x^4+y^4+z^4+xy(x^2+y^2)+yz(y^2+z^2)+zx(x^2+z^2)+xyz(x+y+z) \ge 4(x^2y^2+y^2z^2+z^2x^2) $
Áp dụng bất đẳng thức Schur bậc 4:
$ x^4+y^4+z^4+xyz(x+y+z) \ge xy(x^2+y^2) +yz(y^2+z^2) + zx(z^2+x^2) $
Vậy ta cần chứng minh:
$ xy(x^2+y^2)+yz(y^2+z^2)+x(x^2+z^2) \ge 2(x^2y^2+y^2z^2+z^2x^2) $
Bất đẳng thức trên luôn đúng theo AM-GM.
Hoàn tất chứng minh.


#182
Khoa Linh

Khoa Linh

    Thiếu úy

  • Thành viên
  • 601 Bài viết

Bài 77(APMO 2004): Cho a, b, c là các số thực dương. CMR: $(a^2+2)(b^2+2)(c^2+2)\geq 9(ab+bc+ca)$

Cách 2

Áp dụng Dirichlet ta giả sử $(b^2-1)(c^2-1)\geq 0\Leftrightarrow b^2c^2\geq b^2+c^2-1$

Suy ra $(a^2+2)(b^2+2)(c^2+2)=(a^2+2)(b^2c^2+2b^2+2c^2+4)\geq (a^2+2)(3b^2+3c^2+3)=3(a^2+1+1)(1+b^2+c^2)\geq 3(a+b+c)^2$


$\sqrt[LOVE]{MATH}$

"If I feel unhappy, I do mathematics to become happy. If I am happy, I

 

do mathematics to keep happy" - Alfréd nyi 


#183
Khoa Linh

Khoa Linh

    Thiếu úy

  • Thành viên
  • 601 Bài viết

Bài 79: Cho a, b, c là các số thực dương thỏa mãn $a+b+c=3$. CMR:

$(a^2-a+1)(b^2-b+1)(c^2-c+1)\geq 1$

Cách 1(Dirichlet):

Giả sử $(b-1)(c-1)\geq 0\Rightarrow (b^2-b+1)(c^2-c+1)=bc(b-1)(c-1)+b^2+c^2-b-c+1\geq \frac{(b+c)^2}{2}-(b+c)+1=\frac{(3-a)^2}{2}-(3-a)+1=\frac{a^2-4a+5}{2}$

Vậy ta cần đi chứng minh: 

$(a^2-a+1)(a^2-4a+5)\geq 2\Leftrightarrow (a-1)^2)(a^2-3a+3)\geq 0$ luôn đúng

Cách 2(Cauchy - Schwarz):

Giả sử $c=min\left \{ a,b,c \right \}\Rightarrow c\leq 1$

Áp dụng Cauchy - Schwarz ta có:

$(a^2-a+1)(b^2-b+1)=\left ( (a-\frac{1}{2})^2+\frac{1}{2}+\frac{1}{4} \right )\left ( \frac{1}{4}+\frac{1}{2}+(b-\frac{1}{2})^2 \right )\geq\left [ \frac{1}{2}\left ( a-\frac{1}{2} \right )+\frac{1}{2}\left (b-\frac{1}{2} \right )+\frac{1}{2} \right ]^2=\frac{(3-c)^2}{4}$

Ta cần đi chứng minh: 

$(c^2-c+1)(3-c)^2\geq 4\Leftrightarrow (c-1)^2(c^2-5c+5)\geq 0$ luôn đúng do giả sử $c\leq 1$


$\sqrt[LOVE]{MATH}$

"If I feel unhappy, I do mathematics to become happy. If I am happy, I

 

do mathematics to keep happy" - Alfréd nyi 


#184
Khoa Linh

Khoa Linh

    Thiếu úy

  • Thành viên
  • 601 Bài viết

Bài 82: Cho 3 số dương a, b, c thỏa mãn $a+b+c=3$. Chứng minh rằng: 

$\frac{a}{ab+1}+\frac{b}{bc+1}+\frac{c}{ca+1}\geq \frac{3}{2}$

(Sưu tầm)


Bài viết đã được chỉnh sửa nội dung bởi Khoa Linh: 23-04-2018 - 18:26

$\sqrt[LOVE]{MATH}$

"If I feel unhappy, I do mathematics to become happy. If I am happy, I

 

do mathematics to keep happy" - Alfréd nyi 


#185
tr2512

tr2512

    Thượng sĩ

  • Thành viên
  • 272 Bài viết

bài mạnh hơn bài 77 đây

Bài 80 : Cho a,b,c là các số thực dương

C/m : $(a^2+3)(b^2+3)(c^2+3)\geq 4(a+b+c+1)^2$

Mấy bài đối xứng không thuần nhất coi bộ Dirichlet vẫn là an toàn nhất :D

Không làm giảm tính tổng quát, giả sử $ (a^2-1)(b^2-1) \ge 0 $
$ \Rightarrow a^2b^2 \ge a^2+b^2-1 $
$ \Rightarrow (a^2+3)(b^2+3)(c^2+3) \ge (a^2+b^2-1+3a^2+3b^2+9)(c^2+3) $
Vậy ta cần chứng minh:
$ (4a^2+4b^2+8)(c^2+3) \ge 4(a+b+c+1)^2 $
$ \Leftrightarrow (a^2+b^2+1+1)(1+1+c^2+1) \ge (a+b+c+1)^2 $ (Luôn đúng theo C-S).
Hoàn tất chứng minh.
 
@KhoaLinh: coi lại đề bài 82, hoàn toàn có phản ví dụ đấy :)

Bài viết đã được chỉnh sửa nội dung bởi tr2512: 24-04-2018 - 12:19


#186
mduc123

mduc123

    Hạ sĩ

  • Thành viên
  • 55 Bài viết

Bài 83: Cho a,b,c>0 thỏa mãn ab+bc+ca=1.Tìm giá trị lớn nhất của biểu thức:

$P=\frac{a}{\sqrt{1+a^{2}}}+\frac{b}{\sqrt{1+b^{2}}}+\frac{2c}{\sqrt{1+c^{2}}}$



#187
xuanhoan23112002

xuanhoan23112002

    Trung sĩ

  • Thành viên
  • 103 Bài viết

Bài 84: Cho $0< x, y, z< 1$ thỏa mãn: $xyz=(1-x)(1-y)(1-z)$. CMR: $x^2+y^2+z^2\geq \frac{3}{4}$



#188
xuanhoan23112002

xuanhoan23112002

    Trung sĩ

  • Thành viên
  • 103 Bài viết

Bài 85: Cho a, b, c là các số thực dương thỏa mãn $a^2+b^2+c^2+abc=4$. CMR: 

$a+b+c\geq \sqrt{a}+\sqrt{b}+\sqrt{c}$


Bài viết đã được chỉnh sửa nội dung bởi xuanhoan23112002: 23-04-2018 - 19:56


#189
tr2512

tr2512

    Thượng sĩ

  • Thành viên
  • 272 Bài viết

 

Bài 76: 

Cho 3 số thực a, b, c thỏa mãn $ abc \ge 0$. Chứng minh bất đẳng thức:
$ a^2+b^2+c^2+2abc+4 \ge 2(a+b+c)+ab+bc+ca $ (Lê Khánh Sỹ)

 

Giải\\
Trước hết, ta thấy $ abc(a-1)^2(b-1)^2(c-1)^2 \ge 0 $
Do đó trong 3 hạng tử $c(a-1)(b-1); b(a-1)(c-1), a(b-1)(c-1) $ phải có ít nhất một hạng tử không âm
Không làm giảm tính tổng quát, giả sử đó là $ c(a-1)(b-1) \ge 0$, ta có: $ abc \ge ca+cb-c $
Áp dụng vào bất đẳng thức trên, ta có:
$$ a^2+b^2+c^2+2abc+4 \ge a^2+b^2+c^2+2ca+2cb-2c+4 $$
Vậy ta chỉ cần chứng minh:
$$ a^2+b^2+c^2+2ca+2cb-2c+4 \ge 2(a+b+c)+ab+bc+ca $$
$$ \Leftrightarrow (c+\frac{a+b}{2}-2)^2+\frac{3(a-b^2)}{4} \ge 0 \text{ (Luôn đúng)}$$
Chứng minh hoàn tất.
Bất đẳng thức xảy ra khi $a = b = c = 1$ hoặc $ a=0; b=c=2 $ và các hoán vị

Bài viết đã được chỉnh sửa nội dung bởi tr2512: 24-04-2018 - 12:19


#190
DinhXuanHung CQB

DinhXuanHung CQB

    Trung sĩ

  • Thành viên
  • 118 Bài viết

Bài 74: 

Cho a, b, c là 3 số thực không âm. Chứng minh:

$\frac{a^4}{a^2+ab+b^2}+\frac{b^4}{b^2+bc+c^2}+\frac{c^4}{c^2+ac+c^2} \ge \frac{a^3+b^3+c^3}{a+b+c} $

 

@Momo123: đề ban đầu chắc bị DOTOANANG gõ nhầm, thử (a,b,c) =(0.11; 0.5; 0.4) vào đề lúc đầu thấy VT<VP mà :D

$\sum \frac{(a)^{4}}{a^2+b^2+ab}=\sum \frac{(a)^{6}}{a^4+a^3b+a^2b^2}\geq \frac{(\sum a^3)^{3}}{\sum a^4+a^3b+a^2b^2}$

Ta cần chứng minh $\sum (a^4+a^3b+a^2b^2)\leq (\sum a)(\sum a^3)$

<=> $\sum a^2b^2\leq \sum a^3b$

Ta thấy nếu $\sum a^3b\geq \sum a^2b^2$ thì $\sum ab^3\geq \sum a^2b^2$

Mà $\sum (a^3b+ab^3+a^2b^2)\geq 3\sum a^2b^2$

<=> $\sum (a^3b+ab^3)\geq 2\sum a^2b^2$

Vậy có dpcm

NOTE : SAI RỒI NHÉ CÁC BẠN, MÌNH SẼ CHƯA XÓA LỜI GIẢI VÌ MÌNH ĐANG TÌM CÁCH GIẢI ĐẸP THEO HƯỚNG $SCHWARS$, NẾU KHÔNG TÌM RA MÌNH SẼ XÓA


Bài viết đã được chỉnh sửa nội dung bởi DinhXuanHung CQB: 23-04-2018 - 22:54

Little Homie


#191
tr2512

tr2512

    Thượng sĩ

  • Thành viên
  • 272 Bài viết

$\sum \frac{(a)^{4}}{a^2+b^2+ab}=\sum \frac{(a)^{6}}{a^4+a^3b+a^2b^2}\geq \frac{(\sum a^3)^{3}}{\sum a^4+a^3b+a^2b^2}$

Ta cần chứng minh $\sum (a^4+a^3b+a^2b^2)\leq (\sum a)(\sum a^3)$

<=> $\sum a^2b^2\leq \sum a^3b$

Ta thấy nếu $\sum a^3b\geq \sum a^2b^2$ thì $\sum ab^3\geq \sum a^2b^2$

Mà $\sum (a^3b+ab^3+a^2b^2)\geq 3\sum a^2b^2$

<=> $\sum (a^3b+ab^3)\geq 2\sum a^2b^2$

Vậy có dpcm

Coi lại lời giải đi, cái in đậm này có cả đống phản ví dụ


Bài viết đã được chỉnh sửa nội dung bởi tr2512: 23-04-2018 - 21:41


#192
DinhXuanHung CQB

DinhXuanHung CQB

    Trung sĩ

  • Thành viên
  • 118 Bài viết

Bài 82: Cho 3 số dương a, b, c thỏa mãn $a+b+c=3$. Chứng minh rằng: 

$\frac{a}{ab+1}+\frac{b}{bc+1}+\frac{c}{ca+1}\geq \frac{3}{2}$

(Sưu tầm)

$\frac{a}{ab+1}=a-\frac{a^2b}{ab+1}$

Vậy thì cần cm $\sum \frac{a^2b}{ab+1}\leq \frac{3}{2}$

Theo $AM-GM$ $ab+1\geq 2\sqrt{ab}$ => $\sum \frac{a^2b}{ab+1}\leq \frac{a\sqrt{ab}}{2}$

Như vậy cần cm $\sum a\sqrt{ab}\leq 3=\frac{(a+b+c)^2}{3}$

Ta có bdt $3(\sum x^3y)\leq (x^2+y^2+z^2)^2$ (Vasile)

Như vậy thay $x=\sqrt{a}$ và tương tự thì ta có dpcm


Bài viết đã được chỉnh sửa nội dung bởi DinhXuanHung CQB: 23-04-2018 - 21:55

Little Homie


#193
DinhXuanHung CQB

DinhXuanHung CQB

    Trung sĩ

  • Thành viên
  • 118 Bài viết

Coi lại lời giải đi, cái in đậm này có cả đống phản ví dụ

Sorry nghen, sai rồi để mình xem lại


Little Homie


#194
thanhdatqv2003

thanhdatqv2003

    Trung sĩ

  • Thành viên
  • 159 Bài viết

 

Mấy bài đối xứng không thuần nhất coi bộ Dirichlet vẫn là an toàn nhất :D

Không làm giảm tính tổng quát, giả sử $ (a^2-1)(b^2-1) \ge 0 $
$ \Rightarrow a^2b^2 \ge a^2+b^2-1 $
$ \Rightarrow (a^2+3)(b^2+3)(c^2+3) \ge (a^2+b^2-1+3a^2+3b^2+9) $
Vậy ta cần chứng minh:
$ (4a^2+4b^2+8)(c^2+3) \ge 4(a+b+c+1)^2 $
$ \Leftrightarrow (a^2+b^2+1+1)(1+1+c^2+1) \ge (a+b+c+1)^2 $ (Luôn đúng theo C-S).
Hoàn tất chứng minh.
 
@KhoaLinh: coi lại đề bài 82, hoàn toàn có phản ví dụ đấy :)

 

anh xem lại dòng thứ 4 trên xuống còn thiếu đoạn vp cần viết nhân thêm (c^3+3)


:ohmy: [Không tồn tại các nghiệm nguyên khác không x, y, và z thoả mãn xn + yn = zn trong đó n là một số nguyên lớn hơn 2.  (FERMAT)  :ohmy: 

 

 

 

 


#195
Tea Coffee

Tea Coffee

    Trung úy

  • Điều hành viên THPT
  • 772 Bài viết

Bài 84: Cho $0< x, y, z< 1$ thỏa mãn: $xyz=(1-x)(1-y)(1-z)$. CMR: $x^2+y^2+z^2\geq \frac{3}{4}$

$x+y+z\geq 3\sqrt[3]{xyz}$

$xyz=(1-x)(1-y)(1-z)\leq (\frac{3-x-y-z}{3})^{3}=>3\sqrt[3]{xyz}\leq 3-x-y-z\leq 3-3\sqrt[3]{xyz}$

$=>xyz\leq \frac{1}{8}$


Bài viết đã được chỉnh sửa nội dung bởi Tea Coffee: 23-04-2018 - 23:56

Treasure every moment that you have!
And remember that Time waits for no one.
Yesterday is history. Tomorrow is a mystery.
Today is a gift. That’s why it’s called the present.


#196
thanhdatqv2003

thanhdatqv2003

    Trung sĩ

  • Thành viên
  • 159 Bài viết

Bài 83: Cho a,b,c>0 thỏa mãn ab+bc+ca=1.Tìm giá trị lớn nhất của biểu thức:

$P=\frac{a}{\sqrt{1+a^{2}}}+\frac{b}{\sqrt{1+b^{2}}}+\frac{2c}{\sqrt{1+c^{2}}}$

Ta có: P=$\frac{a}{\sqrt{1+a^2}}+\frac{b}{\sqrt{1+b^2}}+\frac{2c}{\sqrt{1+c^2}}=\sqrt{\frac{a^2}{(a+b)(a+c)}}+\sqrt{\frac{b^2}{(b+a)(b+c)}}+\sqrt{\frac{4c^2}{(c+a)(c+b)}}$

                                                                                                                                                                                                                            (Thay ab+bc+ac=1)

             =$\sqrt{\frac{a}{2(a+b)}.\frac{2a}{a+c}}+\sqrt{\frac{b}{2(a+b)}.\frac{2b}{b+c}}+\sqrt{\frac{2c}{a+c}.\frac{2c}{c+b}}$

Áp dụng AM-GM:

Suy ra : P$\leq \frac{1}{2}(\frac{a}{2(a+b)}+\frac{2a}{a+c}+\frac{b}{2(a+b)}+\frac{2b}{b+c}+\frac{2c}{a+c}+\frac{2c}{c+b})$

                        =$\frac{1}{2}(\frac{1}{2}+2+2)=\frac{9}{4}$

Vậy GTLN của P=$\frac{9}{4}$

P/s: Bài này đã làm qua thật may là vẫn còn nhớ  :D  :D  :D


:ohmy: [Không tồn tại các nghiệm nguyên khác không x, y, và z thoả mãn xn + yn = zn trong đó n là một số nguyên lớn hơn 2.  (FERMAT)  :ohmy: 

 

 

 

 


#197
Korkot

Korkot

    Thượng sĩ

  • Thành viên
  • 227 Bài viết

Bài 86: cho x,y,z,p,q,r thỏa x+y+z=p+q+r=1 và $pqr \leq \frac{1}{2}$
CMR $px+qy+rz \geq 8xyz$


Bài viết đã được chỉnh sửa nội dung bởi MoMo123: 24-04-2018 - 21:29

  Nếu bạn cứ tiếp tục ca thán về cùng một nỗi buồn, cùng một việc nhỏ nhặt, bạn sẽ mãi mãi chìm đắm trong thất bại và sống một  cuộc đời nhỏ bé. Hãy luôn nhớ rằng, ngay cả một ngày tồi tệ nhất cũng chỉ có 24 tiếng đồng hồ mà thôi.

                   :like  :like  :like  :like  :like  :like  :like  :like  :like  :like  :like  :like  :like  :like  :like  :like  :like  :like  :like  :like 


#198
thanhdatqv2003

thanhdatqv2003

    Trung sĩ

  • Thành viên
  • 159 Bài viết

Bài 87 : Cho các số thực dương  x,y,z  thỏa mãn $x^2+y^2+z^2=3$

Chứng minh rằng: $\frac{x}{3-yz}+\frac{y}{3-xz}+\frac{z}{3-xy}$


:ohmy: [Không tồn tại các nghiệm nguyên khác không x, y, và z thoả mãn xn + yn = zn trong đó n là một số nguyên lớn hơn 2.  (FERMAT)  :ohmy: 

 

 

 

 


#199
Tea Coffee

Tea Coffee

    Trung úy

  • Điều hành viên THPT
  • 772 Bài viết

Bài 87 : Cho các số thực dương  x,y,z  thỏa mãn $x^2+y^2+z^2=3$

Chứng minh rằng: $\frac{x}{3-yz}+\frac{y}{3-xz}+\frac{z}{3-xy}$

Chắc là CM $\leq \frac{3}{2}$

Ta có: $P=\frac{x}{3-yz}+\frac{y}{3-xz}+\frac{z}{3-xy}$

$\frac{x}{3-yz}\leq \frac{x}{3-\frac{y^{2}+z^{2}}{2}}=\frac{2x}{6-y^{2}-z^{2}}=\frac{2x}{3+x^{2}}\leq \frac{2x}{2+2x}=\frac{x}{1+x}$

$=>P\leq \frac{x}{1+x}+\frac{y}{1+y}+\frac{z}{1+z}=3-(\frac{1}{1+x}+\frac{1}{1+y}+\frac{1}{1+z})$ $\leq 3-\frac{9}{x+y+z+3}\leq 3-\frac{9}{6}=\frac{3}{2}$


Treasure every moment that you have!
And remember that Time waits for no one.
Yesterday is history. Tomorrow is a mystery.
Today is a gift. That’s why it’s called the present.


#200
DOTOANNANG

DOTOANNANG

    Đại úy

  • ĐHV Toán Cao cấp
  • 1609 Bài viết

Bài 88. Cho $abc\,\geqq 1$. Chứng minh rằng: $(a+ \frac{3}{a+ 5})+ (b+ \frac{3}{b+ 5})+ (c+ \frac{3}{c+ 5})\geqq (\frac{3}{2})^{3}$







Được gắn nhãn với một hoặc nhiều trong số những từ khóa sau: bất đẳng thức, holder, cosi, bunhiacopxki

1 người đang xem chủ đề

0 thành viên, 1 khách, 0 thành viên ẩn danh